mch exam 3

Réussis tes devoirs et examens dès maintenant avec Quizwiz!

A nurse in the ED is caring for an adolescent who has severe abdominal pain due to appendicitis. Which of the following locations should the nurse identify as McBurney's point? (Hotspot)

A. the lower R. quadrant of the abdomen between the umbilicus and the anterior iliac crest. A ​is ​correct. ​The ​nurse ​should ​identify ​the ​lower ​right ​quadrant ​of ​the ​abdomen ​between ​the ​umbilicus and ​the ​anterior ​iliac ​crest ​as ​the ​location ​of ​McBurney's ​point.

Assessing a school-age child immediately postop following a perforated appendix repair. Expected findings?

Absence of peristalsis The ​nurse ​should ​expect ​absence ​of ​peristalsis ​in ​the ​immediate ​postoperative ​period, ​until ​the ​bowel resumes ​functioning.

Preparing an adolescent for lumbar puncture. Appropriate action?

Apply topical analgesic cream to the site 1 hr prior to the procedure. The ​nurse ​should ​apply ​a ​topical ​analgesic ​to ​the ​lumbar ​site ​60 ​min ​prior ​to ​the ​procedure ​to ​decrease the ​adolescent's ​pain ​while ​the ​lumbar ​needle ​is ​inserted.

A nurse is creating a plan of care for a preschooler who has Wilm's tumor and is scheduled for surgery. Which of the following interventions should the nurse include?

Avoid palpating the abdomen when bathing the child before surgery The ​nurse ​should ​avoid ​palpating ​the ​abdomen ​when ​bathing ​the ​child ​before ​surgery ​because movement ​of ​the ​tumor ​can ​cause ​cancer ​cells ​to ​disseminate ​to ​other ​sites, ​adjacent ​and ​distant ​to ​the tumor ​site.

Pediatric Emergencies: Planning Interventions for Lead Exposure (RN QSEN - Safety , Active Learning Template - System Disorder, RM NCC RN 10.0 chp 43)

COMPLICATIONS Cognitive impairments Varies with degree of anoxic insult or lead levels in blood. *NURSING CONSIDERATIONS* ●Provide prevention measures to families. ●Routine screening for lead levels at 1, 2, and 3 years of age. ●Provide case management for children who have elevated lead levels. ●Make appropriate referrals (community nurse, teacher, early intervention).

Immunizations: Determining Possible Contraindications (RM NCC RN 10.0 chp 35, RN QSEN - Safety , Active Learning Template - Basic Concept)

COMPLICATIONS, CONTRAINDICATIONS, AND PRECAUTIONS ●A severe allergic reaction, such as anaphylaxis, can occur in response to any vaccine, and is a contraindication for receiving further doses of that vaccine or other vaccines containing that substance. ●Moderate or severe illnesses with or without fever are precautions to receiving immunizations. ●The common cold and other minor illnesses are not contraindications to immunizations. ●Severe febrile illness is a contraindication to all immunizations. ●Do not administer live virus vaccines, such as varicella or MMR, to a child who is severely immunocompromised. ●Precautions to immunizations require providers to analyze data and weigh the risks that come with immunizing or not immunizing.

A nurse is reviewing the lab report of a school-age child who is experiencing fatigue. Which of the following findings should the nurse recognize as an indication of anemia?

Hematocrit 28% The ​nurse ​should ​recognize ​that ​this ​hematocrit ​level ​is ​below ​the ​expected ​reference ​range ​for ​a school-age ​child. ​The ​child ​can ​exhibit ​fatigue, ​lightheadedness, ​tachycardia, ​dyspnea, ​and ​pallor ​due ​to the ​decreased ​oxygen-carrying ​capacity.

A​ ​nurse​ ​is​ ​creating​ ​a​ ​plan​ ​of​ ​care​ ​for​ ​an​ ​infant​ ​who​ ​has​ ​an​ ​epidural​ ​hematoma​ ​with​ ​a skull​ ​fracture.​ ​Which​ ​of​ ​the​ ​following​ ​actions​ ​should​ ​the​ ​nurse​ ​include​ ​in​ ​the​ ​plan?

Implement​ ​seizure​ ​precautions​ ​for​ ​the​ ​infant. The ​nurse ​should ​implement ​seizure ​precautions ​for ​an ​infant ​who ​has ​an ​epidural ​hematoma ​as ​a ​safety measure.

Burns: Expected Clinical Manifestations for Septic Shock (Active Learning Template - System Disorder, RM NCC RN 10.0 chp 32)

NURSING ACTIONS ●Administer IV crystalloid solutions for the first 24 hr followed by colloid solutions. ●Meticulously monitor I&O. ●Monitor laboratory findings, noting indications of anemia and infection. ●Monitor vital signs. ●Assess sensorium. ●Assess capillary refill in extremities. Monitor for manifestations of septic shock, and notify the provider of findings. ◯Alterations in sensorium (confusion) ◯Increased capillary refill ◯Spiking fever ◯Mottled or cool extremities ◯Decreased bowel sounds ◯Tachycardia ◯Tachypnea ◯Decreased urine output

A nurse is caring for a 10-year-old child following a head injury. Findings that indicate the development of diabetes insipidus?

Sodium 155 mEq/L A ​child ​who ​has ​a ​head ​injury ​can ​develop ​diabetes ​insipidus ​as ​a ​result ​of ​pituitary ​hypofunction ​leading to ​a ​deficiency ​of ​antidiuretic ​hormone. ​Underexcretion ​of ​antidiuretic ​hormone ​leads ​to ​polyuria ​and polydipsia ​and ​possibly ​dehydration. ​With ​the ​excessive ​loss ​of ​free ​water, ​sodium ​levels ​rise ​above ​the expected ​reference ​range.

A nurse is assessing a pt. who has a new diagnosis of celiac disease. Expected clinical manifestations?

Steatorrhea The ​nurse ​should ​realize ​that ​clients ​who ​have ​celiac ​disease ​are ​unable ​to ​digest ​gluten. ​This ​will ​cause damage ​to ​the ​cells ​in ​the ​bowel, ​leading ​to ​malabsorption, ​steatorrhea, ​and ​diarrhea.

A nurse in an ED is performing a physical assessment on a 2-wk-old male infant. Manifestation that is the priority to report to the provider?

Substernal retractions When ​using ​the ​airway, ​breathing, ​circulation ​approach ​to ​client ​care, ​the ​nurse ​should ​determine ​that the ​priority ​finding ​to ​report ​to ​the ​provider ​is ​substernal ​retractions. ​This ​finding ​indicates ​the ​infant ​is experiencing ​acute ​respiratory ​distress ​and ​increased ​respiratory ​effort, ​which ​could ​quickly ​progress ​to respiratory ​failure

A nurse is preparing to suction an infant who has a tracheostomy. Which of the following actions should the nurse take

Suction for 5 seconds or less

A nurse in an ED is assessing a 3-month-old infant who has rotavirus and is experiencing acute vomiting and diarrhea. Manifestations that indicate moderate to severe dehydration?

Sunken anterior fontanel The ​nurse ​should ​recognize ​that ​a ​sunken ​anterior ​fontanel ​is ​an ​indication ​of ​moderate ​to ​severe dehydration ​due ​to ​the ​acute ​loss ​of ​fluid.

A nurse is interviewing the parent of an 18-month-old toddler during a well-child visit. The nurse should identify that which of the following findings indicates a need to assess the toddler for hearing loss

The toddler received tobramycin during a hospitalization 2 weeks ago

A nurse is assessing a school-age child who has appendicitis with possible perforation. The nurse should identify which of the following as a manifestation of peritonitis

abd distention

Neuro - seizures - how do you manage seizures in children?

animal/ bug bite, medicine, all kinds of other random stuff can cause seizures Protect airway POSITIONNG put on ground turn to side.. be sure to protect head/neck safe environment document time, length.. .not much to do to stop it without medicine (MD ORDER) within the hospital. EDUCATE PARENT.. keep them calm and informed **EBP - no longer using tongue depressor *web x* maintain airway have to keep them safe turn them to their side if we can open up there clothing pad area if they're in a bed document time when it happens when its over, behaviors they have when they are coming from that post ictal state, whether they were incontinent or not- what type of sz one sz does not mean that they have epilepsy

Review the milestones of infants to toddlers - you need to be aware of these so if a child has not reached a milestone, what questions should you be asking and when do we alert the MD?

babies grow the fast. rapid growth. from head down and center out. - reach out at 3months. birth weight doubles at 6mths and antierior fontanel -6 to 8 weeks 15months of age -- walking, weight trippled, holding things, seperation anxiety, *web x* role to their belly to there back in 5mts 6mts back to their belly (turtle) sit uncompanied at 8 mts 12 mts walking if they're not babbling by 8 mts-----check the hearing maybe a hearing deficit we do head/chest circumferences until 1 yr of age and we should see an evening from head to chest as a 1:1 for toddler and the chest becomes more lateral bellies become more flatter bc they're up and running around 1. look at family dynamics 2. clinical picture 3. birth hx (premature or on time) - premie has a little delay inactivity 4. anterior fontanels closes from 12-18 mts - anterior fontanels is very small, but stays open for awhile - 24 mts we do not expect to find the fontanel open - 24 mts should be walking around - engaged in or completed their toilet training - hear them speaking in small sentences -should have their teeth 5. posterior fontanel is 6-8 wks bc its very small *need to listen to parents* 1. should not compare the children bc each child is different 2. all children grow not at the same rate but they do have the same steps 3. we should see children growing but some are at a different rate 4. we see children grow taller than others 5. some get heavier than others based on the nutritional intake

daily schedules

bathing, medication, sleep, activity *if the child is bathe in the evening let's keep that going *if they eat 3-7 times a day we need to try to mimic that as well *hospital allow for personal preference in dietary intake we want to embrace that while keeping at a dietary restriction *we allow parents/children to choose when they eat and what they eat

When assessing a school age child for scoliosis, it is important to have the child?

bend forward with the knees straight and the arms dangling. This position allows for any adequate visualization of any asymmetry.

How do we communicate with children

clearly slowing, make sure the child is interactive, get on their level, show examples *web X* 1. explain, in-turn they will understand 2. encourage parents to stay in hospital with child 3. bring there favorite toy/ blanket- TRANSITIONAL OBJECT 4. explain how procedure is going to happen by using there favorite toy 5. nonverbal/ verbal cues- communication 6. be cautious with the words we choose- dont be smiling/ frowning, and say another thing 7. get to their level. dont look down to them, avoid that 8. get bed to their level or squat if needed, so were looking at them 9. we talk slowly, quietly, we dont shout at them, we dont increase our voice volume to make a point *reduce stress which will reduce there pain perception, and make respiratory easier and they will be more cooperative*

A nurse is obtaining a problem-oriented history from a preschool-aged child. The nurse should consider that children form this age group typically can?

describe the symptoms Preschoolers are usually able to describe symptoms to their problems.

What is the glascow coma scare - what are the neuro checks we would do on children and remember the LOC is critical to recognize deteriorating issues.

do loc checks to see if they are improving or deteriorating it is important to know what the baseline is so we can see if they're improving or deteriorating happens after head injury.. or for monitoring nero WE DO NOT LAY A CHILD FLAT AFTER A HEAD INJURY because of ICP INTER-OCCULAR-PRESSURE we watch monitor.. LOC is monitored.. assessments Glascow coma scale is 0 -15.. 15-9 normal, 8-4, *****, 3-0 coma, severe neurological issue

Please be aware of Erikson's theories of Development - why? Because if you do not recognize lags in developmental stages we as nurses can not assist the family in helping this child become a social and productive person.

dont want to be overprotective but not to leanent/ fair either. niether is 100% good need to be a lil over protective, but needs to be a little hands off as well every event might require a different tact. today I need to be a little involved and tomorrow i need to let them go a little based on what the activity is * 3 types of parenting*?? 1.

When performing an otoscopy examination on a 2-year old child, the nurse should pull the pinna

down and back This is the correct technique for straightening an ear canal of a 2 year old because it curves upward.

A nurse is caring for a school-age child who is receiving cefazolin via intermittent IV bolus. The child suddenly develops diffuse flushing of the skin and angioedema. After discontinuing the medication infusion, which of the following medications should the nurse administer first

epi

A nurse is caring for a school-age child who is receiving a blood transfusion. Which of the following manifestations should alert the nurse to a possible hemolytic transfusion reaction

flank pain

Immunocompromised children - who would be a greatest risk for infection? And how do you protection.

handwashing isolation -- FEVER LETHARGY CHANGE. Reyes Syndrome - complicated how? HIV, Cancer, Reyes Syndrome, Autoimmune Rhemoid arthrites CF, Children on sterioids HAND WASHING !!! PPE most important best way to protect immunocompromised no flowers or fresh fruit in ICU ASPRIN do not use apsrin especially with a fever or any time near a fever except with kawasaki sydrome the MD will give RX

A nurse is examining an 18-month-old child ears during a well child visit. Which of the following techniques should the nurse use?

have the parent hold the child securely in their lap. The parents lap is the most secure and safe position for a child.

A nurse is reviewing the laboratory report of a 6-year-old child who is receiving chemotherapy. Which of the following laboratory values should the nurse report to the provider

hgb 8.5

medication administration

if using a liquid measurement make sure you get something that graduates use a syringe for babies to titrate the medication and have the correct dosage if a toddler drinks out of a cup we would need to put it in a cup for them to drink- as long as your sure you're going to get all of the medication want to be cautious about putting it into food bc it will alter the taste never put it into formula bc the baby needs it for substance *can place meds in water and juice (can alter taste a lil)- if they drink it down you have to make sure they are getting the entire dosage* if they put it into a large volume they may not be able to take it all in sometimes for babies we put it into a nipple and its not attached to anything and let them suck it through the nipple ***IM, IV meds- make sure the IV is patent and choose your sight wisely for IM medications *** Choose your tools wisely- right size needle and right size syringe we use rectal meds with kids sometime bc it does absorb well/ quickly and we do not have to do an IM injection if you have a child who does not take meds at all and they just spit everything out

A nurse is reviewing the lumbar puncture results of a school-age child suspected of having bacterial meningitis. Which of the following results should the nurse identify as a finding associated with bacterial meningitis

increased protein concentration

A nurse in the emergency department is assessing a toddler who has Kawasaki disease. Which of the following findings should the nurse expect? (Select all that apply

increased temperature xerophthalmia cervical lymphadenopathy

congenital cardiac cath issues

know pre and post procedures for cardiac cath!!!!!!

web x intro

medical sx nursing and peds is the focus for this exam *if you take care of a adult with a respiratory/ cardiac issue *you can take care of a ped just the same * The difference is how we approach, the limitations and medication administration is very different bc of the titration requirements

What drug is most frequently used for severe or postoperative pain in children.

morphine. titrate *web x* *pain in children* 1. morphine is the opiod of choice 2. nsaids 3. relaxation techniques/ distractions 4. redirect 5.movies/games 6. coloring a picture fear exacerbates pain

Injury prevention in each stage of childhood, think it through please.

most popular concern is motor vehicle accidents of all ages - either they're driving themselves, or they're in a little vehicle or bike or skateboard that can cause them to have a run in with a car - in a car but they are not restrained properly ----->for ati we do not get alot of info about carseats in ati text books ----->look up carseats so we know the age and the range the children should be, the degree the seat is supposed to be placed at based on age will need to know the restraint--- where does the strap sit on the child to prevent choking and to keep them safe

Acute and Infectious Respiratory Illnesses: Caring for a Child Who Has Epiglottitis (RN QSEN - Safety , Active Learning Template - System Disorder, RM NCC RN 10.0 Chp 17)

nuRsInG cARe ●Protect airway. ●Avoid throat culture or using a tongue blade. ●Prepare for intubation. ●Provide humidified oxygen. ●Monitor continuous oximetry. ●Administer corticosteroids, and IV fluids as prescribed. ●Administer antibiotic therapy starting with IV, then transition to oral to complete a 10‑day course, as prescribed. ●Droplet isolation precautions for first 24 hr after IV antibiotics initiated

Know the implications and management of a cardiac cath pre and post procedure.

pedal pulses, cool, pale, monitor vitals o2 sats hold pressure on & ABOVE the site. reinforce dressing do not take off restraints are for procedures only.. may be chemical to protect child maintain IV

A nurse is testing a child for strabismus. Which of the following is the correct technique fro performing this examination?

perform the cover-uncover test This test identifies whether a child has strabismus or nonbinocular vision.

acquired cardiac issues

rhematic fever kiwosachi dz which is difficult to dx - it doesnt present itself as a cardiac issue it presents itself as a rash and flu like symptoms huge amount of aspirin bc it acts as a antiinflammatory and a anticoagulant giving aspirin is under full medical direction

Discipline and parental styles may impact a child while hospitalized. Please know the types of discipline and what is beneficial vs potentially harmful.

safety is #1 reason should begin at good for safety, education time out no isolation no corporal punishment Parents discipline styles change *web x* means to keep children safe -limit activity -limit what they use for safety purposes - limit where they can go - use restraints in a motor vehicle in case they get into an accident *discipline is about education and helping kids make the proper decision and knowing they're consequences if they did not make the right decision and if they were willing to accept those consequences* time-out can be used in hospital setting "redirect" have to have rules in a play room or ppl are going to get hurt *disciplinary tactics that are demeaning* 1. spanking striking yelling 2. name calling 3. wishing they were never born

A nurse is providing teaching to the parents of a toddler about the administration of prescribed eye drops and eye ointment. Instructions to include?

"Administer the eye drops 3 minutes before the ointment." The ​nurse ​should ​instruct ​the ​parents ​to ​administer ​the ​eye ​drops ​first ​and ​then ​wait ​3 ​min ​before administering ​the ​eye ​ointment. ​This ​action ​provides ​adequate ​time ​and ​spacing ​for ​each ​separate medication ​to ​work.

A nurse is providing discharge teaching to the parents of a 6-month-old infant who is postoperative following hypospadias repair with a stent placement. Which of the following instructions should the nurse include in the teaching

"Allow the stent to drain directly into your infant's diaper

Providing discharge teaching to the parents of a 3-month-old infant following cheiloplasty. Appropriate instructions to include?

"Apply a thin layer of antibiotic ointment on your baby's suture line daily for the next 3 days. The ​nurse ​should ​instruct ​the ​parents ​to ​apply ​a ​thin ​layer ​of ​antibiotic ​ointment ​on ​the ​infant's ​suture line ​daily ​for ​3 ​days ​and ​then ​continue ​to ​apply ​petroleum ​jelly ​to ​the ​area ​for ​several ​weeks ​to ​promote healing.

A​ ​nurse​ ​is​ ​providing​ ​teaching​ ​about​ ​car​ ​seat​ ​use​ ​to​ ​the​ ​mother​ ​of​ ​a​ ​6-month-old​ ​infant. Which​ ​of​ ​the​ ​following​ ​statements​ ​by​ ​the​ ​mother​ ​indicates​ ​an​ ​understanding​ ​of​ ​the teaching?

"I should secure the car seat using lower anchors and tethers instead of the seat belt." Lower ​anchors ​and ​tethers, ​or ​the ​LATCH ​child ​safety ​seat ​system, ​should ​be ​used ​to ​secure ​an ​infant's car ​seat ​in ​the ​vehicle. ​This ​system ​provides ​anchors ​between ​the ​front ​cushion ​and ​the ​back-rest ​for ​the car ​seat. ​Therefore, ​if ​this ​system ​is ​available, ​the ​seatbelt ​does ​not ​have ​to ​be ​used.

A nurse is teaching a school-age child who has a new diagnosis of type 1 diabetes mellitus. The nurse should identify which of the following statements by the child as understanding the teaching

"I will give myself a shot of regular insulin 30 minutes before I eat breakfast

A nurse is providing discharge teaching to the parent of an 18-month-old toddler who has dehydration as a result of acute diarrhea. Statement by the parent that indicates and understanding of the teaching?

"I will monitor my child's # of wet diapers" The ​nurse ​should ​teach ​the ​parent ​to ​closely ​monitor ​the ​child's ​number ​of ​wet ​diapers. ​Monitoring ​the number ​of ​wet ​diapers ​per ​day ​is ​the ​best ​way ​for ​the ​parent ​to ​monitor ​adequate ​output ​and ​hydration status.

A nurse is providing discharge teaching to the parent of a school-age child who has undergone a tonsillectomy. Which of the following statements by the parent should the nurse identify as understanding the teaching

"I will notify the doctor if I notice that my child is swallowing frequently

A nurse is teaching the parent of an infant who has a Pavlik harness to treat developmental dysplasia of the hip. The nurse should identify that which of the following statements by the parent indicates an understanding of the teaching

"I will place my infant's diapers under the harness straps

A hospice nurse is caring for a preschooler who has a terminal illness. The father tells the nurse that he cannot cope anymore and has decided to move out of the house. Which of the following statements should the nurse make

"Let's talk about some of the ways you have handled previous stressors in your life

A nurse is providing discharge teaching to the parent of a school-age child who has moderate persistent asthma. Which of the following instructions should the nurse include?

"Pulmonary function tests will be performed every 12-24 months to evaluate how your child is responding to therapy." The ​nurse ​should ​inform ​the ​parent ​that ​her ​child ​will ​need ​pulmonary ​function ​tests ​every ​12 ​to ​24 months ​to ​evaluate ​the ​presence ​of ​lung ​disease ​and ​how ​the ​child ​is ​responding ​to ​the ​current treatment ​regimen. ​As ​children ​grow, ​sometimes ​their ​symptoms ​can ​improve ​or ​decline ​and ​treatment needs ​to ​change ​accordingly.

A nurse is providing teaching to the parent of a school-age child who has oral candidiasis and is to begin taking oral nystatin. Which of the following instructions should the nurse include?

"Shake the medication prior to administration." The ​nurse ​should ​instruct ​the ​parent ​to ​shake ​the ​medication ​prior ​to ​administration ​in ​order ​to ​disperse the ​medication ​evenly ​within ​the ​suspension.

A nurse is teaching a school-age child and his parent about postoperative care following cardiac catheterization. Which of the following instructions should the nurse include

"Wait 3 days before taking a tub bath

A nurse in a provider's office is caring for a school-age child who has varicella. The parent asks the nurse when her child will no longer be contagious. Appropriate response by the nurse?

"When your child's lesions are crusted, 6 days after they appear." The ​nurse ​should ​inform ​the ​parent ​that ​the ​child ​is ​contagious ​1 ​day ​prior ​to ​lesion ​eruption ​and ​until the ​vesicles ​have ​crusted ​over, ​which ​usually ​takes ​about ​6 ​day

A nurse is providing dietary teaching to the parent of a school-age child who has cystic fibrosis. Appropriate statement to make?

"You should offer your child high-protein meals and snacks throughout the day. The ​parent ​should ​provide ​a ​diet ​that ​is ​well-balanced ​and ​high ​in ​protein ​and ​calories. ​Children ​who ​have cystic ​fibrosis ​require ​a ​higher ​percentage ​of ​the ​recommended ​dietary ​allowances ​of ​all ​nutrients ​in order ​to ​meet ​their ​energy ​requirements. ​Children ​who ​have ​good ​nutritional ​intake ​have ​improved ​lung function ​and ​decreased ​risk ​of ​infection.

A nurse is providing anticipatory guidance to the parents of an 8-month-old infant during a well-child visit. Which of the following statements should the nurse make

"Your baby should be able to sit unsupported

A nurse is caring for a hospitalized preschooler. The child's mother is going home for a few hrs while another relative stays with the child. Statement that the nurse should make to explain to the child when her mother will return?

"Your mommy will be back after you eat." Preschoolers ​make ​sense ​of ​time ​best ​when ​they ​can ​associate ​it ​with ​an ​expected ​daily ​routine, ​such ​as meals ​and ​bedtime. ​Therefore, ​the ​child ​comprehends ​time ​best ​when ​it ​is ​explained ​to ​them ​in ​relation to ​an ​event ​they ​are ​familiar ​with, ​such ​as ​eating.

A nurse is teaching the parents of a toddler who has cognitive impairment about toilet training. Instructions to include in the teaching?

"Award the child with a sticker when he sits on the potty chair." The ​child ​with ​a ​cognitive ​impairment ​learns ​through ​shaping ​behaviors. ​The ​parents ​should ​reward ​the child ​for ​sitting ​on ​the ​potty ​chair ​as ​a ​reinforcement ​of ​the ​desired ​behavior ​of ​continence. ​As ​the ​child repeats ​this ​action, ​the ​parents ​can ​gradually ​decrease ​this ​reward ​and ​then ​give ​rewards ​for ​the ​next step ​in ​the ​task, ​such ​as ​voiding ​while ​sitting ​on ​the ​potty ​chair.

Providing teaching to the parents of a preschooler who has heart failure and who is beginning to take digoxin twice daily. Appropriate instructions to include?

"Brush the child's teeth after giving the medication." The ​nurse ​should ​instruct ​the ​parents ​to ​brush ​the ​child's ​teeth ​after ​administering ​digoxin ​to ​prevent tooth ​decay ​caused ​by ​the ​medication, ​which ​comes ​as ​a ​sweetened ​liquid ​to ​enhance ​the ​taste.

A nurse is creating an educational plan to teach parents about protecting their children from sunburns. Instructions to be included?

"Choose a waterproof sunscreen with an SPF of at least 15." The ​nurse ​should ​instruct ​parents ​to ​apply ​a ​waterproof ​sunscreen ​with ​an ​SPF ​of ​at ​least ​15 ​for ​children. The ​parents ​should ​apply ​the ​sunscreen ​prior ​to ​sun ​exposure ​to ​reduce ​the ​risk ​of ​sunburn. Reapply ​every ​2-3 ​hrs

A nurse is providing teaching to the family of a school-age child who has juvenile idiopathic arthritis. Instructions to be included in the teaching?

"Encourage the child to perform independent self-care" The ​nurse ​should ​teach ​the ​family ​the ​importance ​of ​encouraging ​the ​child ​to ​perform ​independent self-care. ​This ​will ​minimize ​the ​child's ​pain ​while ​maximizing ​mobility. ​Encouraging ​and ​praising ​the child's ​efforts ​for ​independence ​will ​also ​increase ​his ​self-esteem

A nurse is teaching the parent of a newborn about ways to prevent SIDS. Instructions to be included?

"Give the infant a pacifier at bedtime." The ​nurse ​should ​inform ​the ​parent ​that ​protective ​factors ​against ​SIDS ​include ​breastfeeding ​and ​the ​use of ​a ​pacifier ​when ​the ​infant ​is ​sleeping.

A nurse is providing teaching to the parent of a preschooler about ways to prevent acute asthma attacks. Which of the following statements by the parent should the nurse identify as understanding the teaching

"I should keep my child indoors when I mow the yard

A nurse in an emergency department is caring for a school-age child who has sustained a superficial minor burn from fireworks on his forearm. Which of the following actions should the nurse take

Use an antimicrobial ointment on the affected area

Which of the following communication techniques is most appropriate fro a nurse to employ during the physical examination of a 10 year old?

Use books and other visual aids to advance the interview

Health Promotion of Infants (1 Month to 1 Year): Car Seat Use (RN QSEN - Safety , Active Learning Template - Growth and Development, RM NCC RN 10.0 Chp 3)

*Care of tHe neWBorn after DisCHarGe* ●Newborn infants should be placed in an federally approved car seat at a 45 degree angle to prevent slumping and airway obstruction. The car seat is placed rear facing in the rear seat of the vehicle and secured using the safety belt. The shoulder harnesses are placed in the slots at or below the level of the infant's shoulders. The harness should be snug and the retainer clip placed at the level of the infant's armpits. ●Instruct parents that their newborn will require a checkup by a provider within 72 hr of discharge. This is especially important for breastfed newborns to evaluate weight and hydration status. *Motor-vehicle injuries* ●Infant-only and convertible infant-toddler car seats are available. ●Infants and toddlers remain in a rear-facing car seat until the age of 2 years or the height recommended by the manufacturer. ●The safest area for infants and children is the backseat of the car. ●Do not place rear-facing car seats in the front seat of vehicles with passenger airbags. ●Infants should not be left in parked cars.

Hospitalization, Illness, and Play: Preschoolers' Understanding of Time (RN QSEN - Patient-centered Care, Active Learning Template - Growth and Development, RM NCC RN 10.0 Chp 10)

*NURSING INTERVENTIONS* ●Teach the child and family what to expect during hospitalization. ●Encourage family members to stay with the child during the hospital experience to reduce the stress. ●Maintain routine as much as possible. ●Encourage independence and choices. ●Explain treatments, procedures, and cares to the child. ●Provide developmentally appropriate activities

Managing Client Care: Identifying Priority Child to Assess (RN QSEN - Patient-centered Care, Active Learning Template - Basic Concept, RM Leadership 7.0 Chp. 1)

*PRIORITY INTERVENTIONS* ●First: Airway ◯Identify an airway concern (obstruction, stridor). ◯Establish a patent airway if indicated. ◯Recognize that 3 to 5 min without oxygen causes irreversible brain damage secondary to cerebralanoxia. ●Second: Breathing ◯Assess the effectiveness of breathing (apnea, depressed respiratory rate). ◯Intervene as appropriate (reposition, administernaloxone ●Third: Circulation ◯Identify circulation concern (hypotension, dysrhythmia, inadequate cardiac output, compartmentsyndrome). ◯Institute appropriate actions to reverse or minimize circulatory alteration. ●Fourth: Disability ◯Assess for current or evolving disability (neurological deficits, stroke inevolution). ◯Implement actions to slow down development of disability. ●Fifth: Exposure ◯Remove the client's clothing to allow for a complete assessment or resuscitation ◯Implement measures to reduce the risk for hypothermia by providing warm blankets and IV solutions and using a heating device if needed *TIME MANAGEMENT* Organize care according to client care needs and priorities What must be done immediately (administration of analgesic or antiemetic, assessment of unstable client)? ●What must be done by a specific time to ensure client safety, quality care, and compliance with facility policies and procedures (routine medication administration, vital signs, blood glucose monitoring)? ●What must be done by the end of the shift (ambulation of the client, discharge and/or discharge teaching, dressing change)? ●What can the nurse delegate? ◯What tasks can only the RN perform? ◯What client care responsibilities can the nurse delegate to other health care team members, such as practical nurses (PNs) and assistive personnel (APs)? Use time‑saving strategies and avoid time wasters. (1.2) ●Good time management ◯Facilitates greater productivity. ◯Decreases work‑related stress. ◯Helps ensure the provision of quality and appropriately prioritized client care. ◯Enhances satisfaction with care provided ●Poor time management ◯Impairs productivity. ◯Leads to feelings of being overwhelmed and stressed. ◯Increases omission of important tasks. ◯Creates dissatisfaction with care provided. Time management is a cyclic process. ●Time initially spent developing a plan will save time later and help to avoid management by crisis. ●Set goals and plan care based on established priorities and thoughtful utilization of resources. ●Complete one client care task before beginning the next, starting with the highest priority task. ●Reprioritize remaining tasks based on continual reassessment of client care needs. ●At the end of the day, perform a time analysis and determine if time was used wisely.

Hospitalization, Illness, and Play: Planning Activities for a School-Age Child (RN QSEN - Patient-centered Care, Active Learning Template - Growth and Development, RM NCC RN 10.0 Chp 10)

*School-age children* ●Games that can be played alone or with another person ●Team sports ●Musical instruments ●Arts and crafts ●Collections *School-age children* ●Provide factual information. ●Encourage the child to express feelings. ●Try to maintain a normal routine for long hospitalizations, including time for school work. h●Encourage contact with peer group.

Psychosocial Issues of Infants, Children, and Adolescents: Identifying Suggestive Manifestations of Maltreatment (RN QSEN - Safety , Active Learning Template - Basic Concept)

*WARNING SIGNS OF ABUSE* ●Physical evidence of abuse ●History of injury incompatible with findings ●Vague explanation of injury ●Other injuries discovered that are not related to the original client concern ●Delay in seeking care ●Multiple fractures at different stages of healing ●Bruising in a nonmobile client ●Caregivers/client report conflicting histories ●Statement of possible abuse from a caregiver or client *nursinG care* ●Identify abuse as soon as possible. Conduct detailed history and physical examination. ●Assess for unusual bruising on the abdomen, back, and buttocks. Document thoroughly with size, shape, and color. Use diagrams to represent location. ●Assess the mechanism of injury, which might not be congruent with the physical appearance of the injury. Many bruises at different stages of healing can indicate ongoing beatings. ●Observe for bruises or welts in the shape of a belt buckle or other objects. ●Observe for burns that appear glove‑ or stocking‑like on hands or feet, which can indicate forced immersion into boiling water. Small, round burns can be caused by lit cigarettes. Document detailed descriptions of all findings. ●Note fractures that have unusual features, such as forearm spiral fractures, which could be caused by twisting the extremity forcefully. The presence of multiple fractures is suspicious. ●Check the child for head injuries. Assess the child's level of consciousness, making sure to note equal and reactive pupils. Monitor for nausea/vomiting. ●The nursing priority is to have the child removed from the abusive situation. ●Mandatory reporting is required of all health care providers, including suspected cases of child abuse. There are civil and criminal penalties for not reporting. ●Clearly and objectively document information obtained in the interview and during the physical assessment. ●Photograph and detail all visible injuries. ●Conduct the interview with the client and parents individually. ●Be direct, honest, and professional. ●Use language the child understands. ●Be understanding and attentive. ●Client circumstances are case‑sensitive, and referrals are made to always keep the client safe. When applicable, explain the process if a referral is made to child or adult protective services. ●Assess safety and reduce danger for the victim. ●Use open‑ended questions that require a descriptive response. These questions are less threatening and elicit more relevant information. ●Provide support for the child and parents. ●Demonstrate behaviors for child‑rearing with the parents and child. ●Provide consistent care to the child. ●Avoid asking the child probing questions ●Promote self‑esteem. ●Assist with alleviating feelings of shame and guilt. ●Assist the child with grieving the loss of parents, if indicated. ●Discharge can begin once legal determination of placement has been decided.

Renal Disorders: Assessing Peripheral Edema in a Child Who Has Nephrotic Syndrome (Active Learning Template - Nursing Skill, RM NCC RN 10.0 Chp 26)

*nURsinG CaRe* ●Provide rest. ●Monitor I&O. Monitor urine for protein. ●Monitor vital signs. ●Monitor daily weights; weigh the child on the same scale with the same amount of clothing. ●Monitor edema and measure abdominal girth daily. Measure at the widest area, usually at or above the umbilicus. Assess degree of pitting, color, and texture of skin. ●Monitor and prevent infection. ◯Assist the client to turn, cough, and deep breathe to prevent pulmonary involvement. ◯Monitor vital signs, especially temperature, for changes secondary to infection. ◯Maintain good hand hygiene. ◯Administer antibiotic therapy as prescribed. ●Encourage nutritional intake within restriction guidelines. Salt can be restricted during the edematous phase. ●Cluster care to provide for rest periods. ●Assess skin for breakdown areas. ◯Avoid use of urinary collection bags in very young children. ◯Pad bony prominences, or use a specialty mattress to reduce breakdown of skin. ◯Encourage frequent turning and repositioning. ◯Keep the client's skin dry. ◯Elevate edematous body parts. PHYSICAL ASSESSMENT FINDINGS ●Weight gain over a period of days or weeks ●Facial and periorbital edema: decreased throughout the day ●Ascites ●Edema to lower extremities and genitalia ●Anorexia ●Diarrhea ●Irritability ●Lethargy ●Decreased frothy urine ●Blood pressure within expected reference range or slightly below Corticosteroid: prednisone NURSING CONSIDERATIONS ●60 mg/m2/day for 4 to 6 weeks followed by 40 mg/m2every other day for 2 to 5 months with taper. ●Monitor for adverse effects such as hirsutism, slowed linear growth, hypertension, GI bleeding, infection, and hyperglycemia. ●Administer with meals.CLIENT EDUCATION ●Educate the client and family to avoid large crowds (to decrease the risk of infection). ●Inform the client and family that using corticosteroids can increase appetite, cause weight gain (especially in the face), and cause mood swings. ●Educate the client and the family on the medication regime. ●Educate the client and the family on adverse effects and when to notify the provide

caring for children with a hearing/ vision deficit

*web X* pics/ boards/ ipad sign language turn on lights makes noise when walking in if its a visual interpreter if a language issue face them when we speak to them to get attention - start convo with kids, talk to parents too, but mostly with the kids

Pain is a frequent assessment done on all patients - what about pain assessments in children? What is different, and what are the myths and fallacies re: pain management in children

*web x* -pain is expressed differently in different ppl - parents can tell us if this is a real pain a child is feeling or if this is a behavioral issue - are they withdrawn bc they're angry or upset, or scared, or are they in pain --------- children experience pain 1. ask child about pain 2. assess it by - flacc (toddlers) - faces (3 and up) -crying scale (infants) -numerical scale (10-12) all nurses have to use the same scale!!!!!!! if you start with that scale, it has to be used throughout pt care

Restraints - are used for procedures to keep children safe. Be knowledgeable of the different types of restraints that may be used and why and the nursing interventions to keep them safe.

*web x* 1. procedures only we have physical restraints and chemical restraints. We do not dictate who gets the chemical restraint *physical restrains if needed* (nurse) - starting iv -foley -trach care -stitches *chemical restrains if needed* - after cardiac cath, we need chemical - sedation from over using/ over exerting themselves if a child has cerebral palsy or muscular dystrophy and he is in a wheelchair we have to restrain them in the chair bc he is not slipping and falling and thats when we want to be checking on the restraints

know diabetes

*web x* kids can be active with diabetes they just have to alter there dietary intake we want the kids to be in charge of there care as early as they can bc as soon as they become independent the better monitoring they will do when parents take over the care and dictate food activity and insulin children will never learn to be independent its better if kids learn the dz and become independent asap

Play is a huge component of children's communication and work. Note the types of play, which play form is most relative to each stage of childhood, and how can you incorporate that into your communication and education of the hospitalized child? (GO back to CHAPTER 4)

*web-x* goes with chronological order solitary play (infants) parallel play (toddlers) onlooker play- if it is consistent it becomes a red flag bc it becomes a social problem: -autism - physical disability the child doesnt want ppl to know about - shy - cant hear - parents telling them they're not good enough leading to dev. delays/level associative cooperative (school age & adolescents) more organized and competitive organized *pts who are active are healthy *pts who are inactive are sick maybe a temp inactivity, but we need to recognize this as well if a toddler is taking into a playroom and sits there and stares, is it bc there are afraid, or unfamiliar with the area, or indeed they do not want to participate *we communicate with play* 1. we have them draw pictures 2. color 3. paint a picture 4. build something for us 5. read to us choosing what type of play lets us know where they are at intellectually & dev. PLAY SHOULD BE APART OF CARE PLANNING

A nurse is reviewing the dietary choices of an adolescent who has iron deficiency anemia. The nurse should identify that which of the following menu items has the highest amount of iron?

1/2 cup of raisins The ​nurse ​should ​encourage ​the ​adolescent ​to ​eat ​raisins ​because ​they ​contain ​the ​highest ​amount ​of nonheme ​iron.

A nurse is preparing to administer ibuprofen 5 mg/kg every 6 hr PRN for temps. above 38.0C​ ​(100.5​F) to an infant who weights 17.6 lbs. The infant has a temp. of 38.4C (101.2​F). Available is ibuprofen liquid 100 mg/5mL. How many mL should the nurse administer to the infant per dose? (Nearest whole #)

2 mL

Receiving change-of-shift report on 4 children. Which should be assessed 1st?

A toddler who has a concussion and an episode of forceful vomiting When ​using ​the ​urgent ​vs. ​nonurgent ​approach ​to ​client ​care, ​the ​nurse ​should ​assess ​this ​child ​first. ​An episode ​of ​forceful ​vomiting ​is ​an ​indication ​of ​increased ​intracranial ​pressure ​in ​a ​toddler ​who ​has ​a concussion.

A nurse in an ED is caring for a school-age child who is experiencing an anaphylactic rxn. Priority action by the nurse?

Administer IM epinephrine to the child. When ​using ​the ​urgent ​vs ​nonurgent ​approach ​to ​client ​care, ​the ​nurse ​determines ​that ​the ​priority ​action is ​administering ​IM ​epinephrine ​to ​the ​child. ​During ​an ​anaphylactic ​reaction, ​histamine ​release ​causes bronchoconstriction ​and ​vasodilation. ​This ​is ​an ​emergency ​because ​ultimately ​it ​causes ​decreased ​blood return ​to ​the ​heart.

A nurse is caring for a preschooler who is scheduled for hydrotherapy tx for wound debridement following a burn injury. Appropriate action to take prior to the procedure?

Administer an analgesic to the child. Hydrotherapy ​for ​debridement ​of ​a ​wound ​is ​an ​extremely ​painful ​procedure ​which ​requires ​analgesia and/or ​sedation. ​When ​pain ​is ​controlled, ​it ​leads ​to ​reduced ​physiological ​demands ​on ​the ​body ​caused by ​stress ​and ​decreases ​the ​likelihood ​of ​children ​developing ​depression ​and ​post-traumatic ​stress disorder

A nurse is creating a plan of care for a toddler who has minimal change nephrotic syndrome (MCNS) and 3+ pitting edema. Interventions to be included in the plan of care?

Administer corticosteroids to the toddler. The ​nurse ​should ​recognize ​that ​corticosteroids ​are ​the ​treatment ​of ​choice ​for ​providers ​caring ​for children ​who ​have ​MCNS. ​Therefore, ​the ​nurse ​should ​include ​administration ​of ​prescribed corticosteroids ​in ​the ​plan ​of ​care ​for ​this ​toddler.

A nurse is caring for a 2-week-old infant who is breastfeeding and requires a heel stick. Which of the following actions should the nurse take to minimize the infant's pain

Administer sucrose to the infant prior to the procedure

A nurse is preparing to administer an immunization to a 4-yo child. Which of the following actions should the nurse plan to take?

Administer the immunization using a 24-gauge needle The ​nurse ​should ​administer ​an ​immunization ​for ​a ​4-year-old ​child ​using ​a ​24-gauge ​needle ​to ​minimize the ​amount ​of ​pain ​experienced ​by ​the ​toddler.

Burns are more significant. Know 1

Always airway first, Fluids, Pain SUNSCREENS protect from the sun 1st Degree Burn - partials thickness 2nd Degree - 3rd Degree - Full thickness.. burn through skin In a severe burn what is the acute management and then the risks during the healing process? Airways, Fluids (replaced by weight), pain (morphine is most popular.. titrate until 50kg), infection HAVE NARCAN AVAILABLE Long term management, risks? debridement, skin grafting, body image!!! muscles can pull causing issue... so we need Passive ROM and extremities in position of use! tissue not touching tissue to prevent fusing

Always maintain an airway first....

Always maintain an airway first....

A nurse is performing hearing screenings for children at a community health fair. Which of the following children should the nurse refer to a provider for a more extensive hearing evaluation?

An infant who is 8 months old and is not yet making babbling sounds. The ​nurse ​should ​refer ​an ​infant ​who ​is ​not ​making ​babbling ​sounds ​by ​the ​age ​of ​7 ​months ​to ​a ​provider for ​more ​extensive ​evaluation ​of ​hearing.

A nurse is caring for a toddler who has spastic (pyramidal) cerebral palsy. Expected clinical manifestations?

Ankle clonus Exaggerated stretch reflexes Contractures

A school nurse is assessing an adolescent who has scoliosis. Which of the following findings should the nurse expect

a unilateral rib hump

Assessment: ​RN ​Nursing ​Care ​of ​Children ​Online ​Practice ​2016 ​A

Assessment: ​RN ​Nursing ​Care ​of ​Children ​Online ​Practice ​2016 ​A

Assessment: ​RN ​Nursing ​Care ​of ​Children ​Online ​Practice ​2016 ​A remediation

Assessment: ​RN ​Nursing ​Care ​of ​Children ​Online ​Practice ​2016 ​A remediation

A school nurse is assessing a school-age child's BP while he is seated in a chair. The child starts to experience a tonic-clonic seizure. Appropriate action to take first?

Assist the child to a side-lying position on the floor The ​greatest ​risk ​to ​this ​child ​is ​aspiration, ​occlusion ​of ​the ​airway, ​and ​bodily ​injury ​from ​falling ​out ​of ​the chair. ​The ​nurse ​should ​ease ​the ​child ​down ​to ​floor ​in ​a ​side-lying ​position ​immediately. ​This ​position enables ​the ​child's ​secretions ​to ​drain ​from ​the ​mouth, ​preventing ​aspiration, ​and ​maintaining ​a ​patent airway.

A nurse is providing teaching about social development to the parents of a preschooler. Which of the following play activities should the nurse recommend for the child?

Playing dress-up The ​nurse ​should ​instruct ​the ​parents ​that ​at ​the ​preschool ​age, ​play ​should ​focus ​on ​social, ​mental, ​and physical ​development. ​Therefore, ​playing ​dress-up ​is ​a ​recommended ​play ​activity ​for ​this ​child.

A community health nurse is assessing an 18-month-old toddler in a community day care. Which of the following findings should the nurse identify as a potential indication of physical neglect

Poor personal hygiene

Nutrition is the single most important factor in the growth and development of children - so we know that nutrition is a concept that is seen throughout - and should be focused on. Toddlers particularly are picky eaters - so they may incur physiological anorexia and physiological anemia due to the milk ingestion. Are food fads that different children encounter harmful? No, and are usually self- limiting. Adolescents have many different needs for greater caloric intake and more concentrated iron, folic acid, (Bvit) and protein. You will need to understand those differences as well as discuss eating disorders.

Breast milk or formula until 1year of age.. no other milk!!! solid foods start around 6months.. rice cereal, iron fortified, choking with toddlers is a real concerns.. they are also picky eater. which is not a problem unless losing weight. *web-x* infancy & adolescents is the greatest rate of growth (physiologically & cognitively) in between those we slow growing until adolescents *infant nutritional needs* 1. require breast milk and if they do not get breast milk then they get formula 2. they do not get any type of cows milk or table food to atleast 6mts- 1 yr (cows milk not until 1 yr) 3. feeding is frequent with low volume bc there so little, but as they grow they can handle more volume with less frequency 4. 6mts we start with solid foods like rice (less allogenic), vegetables, fruits--- add vegetable b4 the fruit--add less sweet 1st 5. add meat last bc it is hardest to digest 6. children start to dev. teeth 7. teeth erupt around 6mts of age 8. you can make your own baby food, grind up what you have *toddler nutritional needs* 1. allergies 2. picky eaters 3. cant sit very long 4. grays?? 5. PHYSIOLOGICAL ANOREXIA they dont eat bc they are to busy 6. PHYSIOLOGICAL ANEMIA too much milk bc that is blocking the iron absorption 7. once the kid is starting to eat solid food they do not need as much milk intake 8. milk is more filling bc once it hits ur gut it becomes a hydrochloric acid and fills u 9. breast milk by this time is not for nutrition it is for comfort *School age & pre-school??* caloric intake like an adult 1800 cal bc so active 6 meals a day bf snack, lunch snack, dinner snack *adolescents* higher protein iron (men for muscle mass) (women menstrual cycle) anemia -fatigue -headaches -infections

A nurse in the ED is caring for a toddler who has partial-thickness burns on his right arm. Which action should the nurse take?

Cleanse the affected area with mild soap and water. The ​nurse ​should ​wash ​the ​affected ​area ​with ​mild ​soap ​and ​water ​to ​remove ​any ​loose ​tissue ​that ​could cause ​infection.

Cardiac system - issues are congenital defects as well as acquired. Acquired are usually associated with an infectious process- what bacteria is the issue? How do we manage.

Congenital = born with it Cyantoic are much worse.. because circulation is so poor. tissue starving of oxygen = immediate management = protect important organs tripod, knees to chest. these usually have to be surgical corrected otherwise people die most common tertralogy of fallot... children will have respiratory issues. fatigue = no exercise Acyantoic kids.. manage self they get tired and sleep Acquired cardiac issues - strepoccocal (bacteria) bug sets into heart mitral values prophylactic antibiotics for everything *web x* *acyonotic children*- have more respiratory issues turn blue throughout there entire bodies -blue chest, lips circumoral lip change -may not have to be repaired sx (initially) as long as they're growing and able to eat - puts themselves into a tripod position bc of fatigue they play then get tired then play again - have more URI bc of the stress bc of the additional blood flow to the lungs/ pulmonary system is now affecting - sleep more *cyonotic congenetial issue*- children turn blue with activity eating crying - cardiac cyonotic heart defect has to be repaired sx - sx once the child is bigger so they can manage their anesthesia better and manage meds better - can communicate if you do sx on a baby you rely more on vitals and behavior

A nurse is providing anticipatory guidance to the parents of a 2-week-old infant about risk factors for sudden infant death syndrome (SIDS). Which of the following risk factors should the nurse include in the teaching

Covering the sleeping infant with a blanket

Problems that may be significant to certain ages - Colic, Ger/Gerd, Crohn's disease, Hirschsprungs' disease, pleas recognize and know the nursing management.

Crohn's disease -- long term steriod use.. Hirschsprungs' disease - causes severe obstruction, pleas recognize and know the nursing management. pyloric stenous = projectile vomiting over feeding cause it to come back out .. projectile poop once a day at least !!! *web x* *different gi problems based on age* infants- colic, gerd toddlers- constipation preschool- constipation due to anxiety adolescents- chrons dz *intervention* for constipation increase fiber and fluid inatake do not give harsh laxative to the kids

A nurse is assessing a 4-year-old child at a well-child visit. Which of the following developmental milestones should the nurse expect to observe

Cuts a shape using scissor

A nurse is assessing a school-age child who has an infratentorial brain tumor. Which of the following findings should the nurse identify as a manifestation of increased intracranial pressure?

Decreased attention span

A nurse is caring for a school-age child who has primary nephrotic syndrome and is taking prednisone. Following 1 wk of tx, which of the following clinical manifestations indicates to the nurse that the medication is effective?

Decreased edema A ​child ​who ​has ​nephrotic ​syndrome ​can ​experience ​edema ​due ​to ​the ​increased ​glomerular permeability, ​which ​increases ​protein ​loss. ​Prednisone ​decreases ​glomerular ​permeability, ​which ​causes fluid ​to ​shift ​from ​the ​extracellular ​spaces, ​decreasing ​edema.

A nurse is caring for a school-age child who has diabetes mellitus and was admitted with a diagnosis of diabetic ketoacidosis. When performing the respiratory assessment, which of the following findings should the nurse expect

Deep respirations of 32/min

A charge nurse is preparing to make a room assignment for a newly admitted school-age child. Which of the following considerations is the nurse's priority when making a room assignment

Disease process

A nurse is monitoring the oxygen saturation level of an infant using pulse oximetry. The nurse should secure the sensor to which of the following areas on the infant

Great toe

Cognitively impaired children of course require more vigilance, more discussions, more education and we must help parents realize their true potential realistically, but the children need to socialize - most important to work through the Developmental stages.

Do not show pain the same, different ways of showing and talking. early recognition !!! do not label listen to parents *web x* all about early intervention we need to recognize a problem early pay attention to what the parents say. they may not be experts they may not be correct in what there dx is but we need to do our assessment and encourage the physicians to put things in order start to do the assessment if they think there is a deficiency autism is a very multi-spectral problem and children dont appear the same if someone thinks they have some delays some communication issues or hearing issues we need to intervene kids at school who dont participate may be having a hearing or visual problem we need to look at those things we dont say they're bad kids dont label children. pay attention to what ppl are doing

A nurse is caring for a toddler who has acute otitis media and a temperature of 40º C (104º F). After administering acetaminophen, which of the following actions should the nurse plan to take to reduce the toddler's temperature

Dress the toddler in minimal clothing

A nurse at an urgent care clinic is assessing an adolescent client who has an upper respiratory tract infection. Which of the following findings should the nurse recognize as a manifestation of pertussis?

Dry, hacking cough The ​nurse ​should ​recognize ​that ​a ​dry, ​hacking ​cough ​is ​a ​manifestation ​of ​pertussis. ​This ​disease ​usually begins ​with ​indications ​of ​an ​upper ​respiratory ​tract ​infection, ​which ​includes ​a ​dry, ​hacking ​cough ​that ​is sometimes ​more ​severe ​at ​night.

Pediatric Emergencies: Manifestations to Report (RN QSEN - Safety , Active Learning Template - Basic Concept, RM NCC RN 10.0 chp 43)

EARLY INDICATIONS OF RESPIRATORY DISTRESS ●Restlessness ●Tachypnea ●Tachycardia ●Diaphoresis ●Nasal flaring ●Retractions ●Grunting ●Dyspnea ●Wheezing

A nurse is admitting a 4-month-old infant who has heart failure. Which of the following findings is the nurse's priority? (Click on the "Exhibit" button for additional information about the client. There are three tabs that contain separate categories of data

Episodes of vomit

A nurse is reviewing laboratory results of a school-age child who is 1 week postoperative following an open fracture repair. Which of the following values should the nurse identify as an indication of a potential complication

Erythrocyte sedimentation rate 18 mm/hr

A nurse is caring for a school-age child who has experienced a tonic-clonic seizure. Which of the following actions should the nurse take during the immediate postictal period

Place the child in a lateral position

A nurse is caring for an infant who is receiving IV fluids for the treatment of Tetralogy of Fallot and begins to have a hypercyanotic spell. Which of the following actions should the nurse take

Place the infant in a knee-chest position

A nurse is discussing organ donation with the parents of a school-age child who has sustained brain death due to a bicycling accident. Which of the following actions should the nurse take first

Explore the parents' feelings and wishes regarding organ donation

Providing anticipatory guidance to the mother of a toddler. Expected behavior characteristics of the toddler to be included in the teaching?

Expresses likes and dislikes The ​nurse ​should ​teach ​the ​mother ​that ​her ​toddler ​will ​begin ​to ​express ​her ​likes ​and ​dislikes. ​This ​is ​the time ​in ​life ​when ​a ​toddler ​is ​developing ​autonomy ​and ​self-concept. ​She ​will ​try ​to ​assert ​herself ​and frequently ​refuse ​to ​comply. ​The ​parent ​should ​allow ​the ​child ​to ​have ​some ​control ​but ​also ​set ​limits ​in order ​for ​her ​to ​learn ​from ​her ​behavior ​and ​learn ​to ​control ​her ​actions.

A nurse is assessing the pain lvl of a 3-yr-old toddler. Appropriate pain assessment scale to use?

FACES pain rating scale. The ​nurse ​should ​use ​the ​FACES ​pain ​rating ​scale ​for ​pediatric ​clients ​who ​are ​3 ​years ​old ​and ​older. ​This scale ​allows ​the ​toddler ​to ​point ​to ​the ​face ​that ​depicts ​the ​current ​level ​of ​pain. ​The ​nurse ​can ​then determine ​the ​need ​for ​pain ​management.

A school nurse is assessing a school-age child who has erythema infectiosum (fifth disease). Which of the following findings should the nurse expect

Facial rash

Constipation may also occur due to increase in activity, anxiety, or disease - so we must get some historical information - what would you be asking?

Fluids are a problem... be cautious concerned and monitor. let child find their way. Let it rain.. poop that is. if it continues for 24-72hrs culture and MD Give them fluid replacement !!!! monitor their cardiac. and infection if we cannot do oral give it in vein IV double check dose calc *web x* toddlers- constipation preschool- constipation due to anxiety adolescents- chrons dz *intervention* for constipation increase fiber and fluid inatake do not give harsh laxative to the kids

Creating a plan of care for a newly-admitted adolescent who has bacterial meningitis. How long should the nurse plan to maintain the adolescent in droplet precautions?

For 24 hr following initiation of antimicrobial therapy The ​nurse ​should ​plan ​to ​maintain ​the ​adolescent ​on ​droplet ​precautions ​for ​at ​least ​24 ​hr ​following initiation ​of ​antimicrobial ​therapy. ​This ​practice ​will ​ensure ​that ​the ​adolescent ​is ​no ​longer ​contagious, which ​protects ​family ​members ​and ​the ​personnel ​caring ​for ​the ​client. ​Prophylactic ​antibiotics ​might ​be prescribed ​to ​individuals ​who ​were ​in ​close ​contact ​with ​the ​adolescent.

A nurse in an emergency department is caring for a school-age child who has appendicitis and rates his abdominal pain at 7 on a 0 to 10 scale. Which of the following actions should the nurse take

Give morphine 0.05mg/kg IV

A nurse is planning care for a preschooler who has cystic fibrosis. Which of the following interventions should the nurse include in the plan

Increase fat content in the child's diet to 40% of total calories

A nurse is assessing an 8-year-old child who has early indications of shock. After establishing an airway and stabilizing the child's respirations, which of the following actions should the nurse take next

Initiate IV access

A nurse is admitting a school-age child who has pertussis. Which of the following actions should the nurse take

Initiate droplet precautions for the child

A nurse is planning care for a school-age child who is in the oliguric phase of acute kidney injury (AKI) and has a sodium level of 129 mEq/L. Which of the following interventions should the nurse include in the plan

Initiate seizure precautions for the child

Safe Administration of Medication: Actions to Reduce a Child's Pain During Immunization (Active Learning Template - Nursing Skill, RM NCC RN 10.0 Chp 8)

Intravenous Assess venipuncture site per facility protocol and prior to administration of medications *Peripheral venous access devices* ●Use a 24- to 20-gauge catheter. ●Use for continuous and intermittent IV medication administration. ●Short-term IV therapy can be completed at home with the assistance of a home health nurse. *NURSING INTERVENTIONS* medication adminiStration ●Calculate the safe dosage for medication. ●Notify the provider if medication dosage is determined to be outside the safe dosage range, and for any questions about medication preparation or route. ●Double-check high-risk and facility-regulated medications with another nurse. ●Use two client identifiers prior to administration: client name and date of birth. Use parent(s) for verification of infants or nonverbal children. Two identifiers from the ID band must be confirmed: client name, date of birth, or hospital identification number. ●Determine parental involvement with administration. ●Allow the child to make appropriate choices regarding administration (choosing the left or right leg, whether the parent or nurse will administer the medication). ●Prepare the child according to age and developmental stage

A nurse is caring for a toddler who is experiencing acute diarrhea and has moderate dehydration. Which of the following nutritional items should the nurse offer to the toddler

Oral rehydration solution

Skin issues- children get many bites, rashes, acne, and many can be treated with over the counter medications and avoid the irritant. Burns are more significant. Know 1st, 2nd, 3rd degree burns and how to manage a child brought into to you. Always airway first. In a severe burn what is the acute management and then the risks during the healing process?

It happens... it happens a lot. just a little different at each age! Close social interactions. poor hygiene. use topical first. short term steroid = ok. *web x* babies have diaper rashes/ newborn rashes adolescents have acne kids in between have bug bites- gets rashes and lacerations from things they encounter outside impetigo and ringworm, lice,varicella,chicken pox, measles mumps hives hive bc they come into contact with something or they are just anxious about something skin issues are unique for different ages and for different issues we need to assess and be cautious not to touch alot of things *the biggest complaint is itching- give something for the itch- we dont want it to itch so much we break open the skin causing an infection* *burns* severe burns hypovolemia is a huge problem and if you have hypovolemia you have renal failure want to protect that volume, might need to give huge volumes of fluid back to help maintain organ support to prevent the compensation once we manage the volume we want to take care of pain management - risk of infection - risk of psychosocial / psychological issues of the scarring depending how severe the burns were - we want to protect children from burns at all cost - educating parents to help them protect there kids from burns - never have them outside without sunscreen on (minor burns)

What will ease their stress - remember each stage has a different fear as well as the 3 basic fears (example - Infants from 6 mos. through toddler hood - fear of separation)

Let parents stay expalin something to them toy blanket choices *web X* *Infants from 6 mos* - doesn't care who is taking care of them, just wants needs to be met (physical, emotional) - recognize who ind. is and starts to become attached to that ind. that has been providing basic care - we need to respect separation anxiety - separation anxiety 6-18 mts *temperament issues, difficult child, slow to warm child separation anxiety is 24 mts or longer *cognitive impairment and disability s.a goes on longer bc parents are a little more protective and keeps children close. * allow parent to participate in much of the care as they choose too, or are able to do * parents may sit back and expect the nurse to do most of it bc you're the nurse AS A CONSUMER GOING INTO A HOSPITAL SETTING PPL JUDGE - parents assume hcp is judging bc their child is ill and they didnt provide good care for the child - parents tend to hang back, so we need to invite them and empower them to let them know we want them to participate - we want parents to tell us whats going on with the child *object permanence is a huge milestone for children to accomplish has nothing to do with stress* "peek a boo" 9-10 mts of age significance of object permanence is going from thinking something is disappeared and know that they're not dissapeared to knowing when they are coming back huge milestone for children not to say that separation anxiety doesn't isn't there its just not as pronounced them if they know that your there still have parents stay in a hospital setting when a child has been admitted for a given period of time NOT PARENTS RESPONSIBILITY TO CARE FOR THE CHILD- WE INVITE THEM TO HELP WITH THE CHILD WE EMPOWER PARENTS, BUT IF THEY CHOOSE TO STEP BACK WE DONT PUT ANY FEELING OUT THERE THAT UR A BAD PARENT BC THEY DONT WANT TO PARTICIPATE *NURSES JOB TO PROVIDE CARE AND SAFETY*

A nurse is assessing an infant who has a ventricular septal defect. Which of the following findings should the nurse expect

Loud, harsh murmur

A nurse is caring for a school-age child who has acute rheumatic fever. Which of the following actions should the nurse take

Maintain the child on bed rest

Medication administration and specific nursing treatments. (use other resources)

Make sure to measure dosage and base it on the weight! double check if needed. after 50kg stop titration considered adult PO 30mins after food.. do not put the medicine in the food or FORMULA

Musculoskeletal - sprains vs fracture - nursing care and observation. (remember RICE)

May need to immobilize with cast, may have pain or issues at the site. how to protect cast.. no water bag shower.. nothing down cast. no denting which can cause _______ check extremities for Circulation if open fracture with infection may need to deal with infection 1st Keep kids mobile and active as possible with restrictions Immobilizing a child is a huge problem and can cause developmental regression.. bring in specialists EVERY SYSTEM OF THE BODY IS AFFECTED BY IMMOBILIZATION in a bad way!! Child life specialist helps interact with child in a therapeutic way (play) to teach them about procedures and the illness. increase understanding too much calcium keeps body from absorbing iron. protein ! *web x* we initially both the same rest ice compress elevate then immobilize those extremities as well *fx* we only know there is a fx bc we do an xray to identify if there is a fx and if there is a fx then there will be a more mobilize formulation process there are different kinds of casts we manage casts by keeping them dry do not use hair dryer it can heat up the material and burn them - do not put anything inside the casts - pay attention for infection - look at the distal extremity for circulation, edema, sensation if they can feel/ move of they have pulses - check cap refill for circulation. we will do this regardless of where the cast is at - encourage rom if app. esp non affected side bc if immobilize a child and put them in a bed or wheelchair they're not using either leg so we want to make sure they're using rom on unaffected leg so they dont have muscle atrophy -

A nurse is caring for a 15-year-old pt following a head injury. Which of the following findings should the nurse identify as an indication that the child is developing syndrome of inappropriate anitidiuretic hormone secretion (SIADH)?

Mental confusion A ​child ​who ​has ​a ​head ​injury ​can ​develop ​SIADH ​as ​a ​result ​of ​altered ​pituitary ​function, ​leading ​to ​an oversecretion ​of ​antidiuretic ​hormone. ​Oversecretion ​of ​antidiuretic ​hormone ​leads ​to ​a ​decrease ​in ​urine output, ​hyponatremia, ​and ​hypoosmolality ​due ​to ​overhydration.

A nurse in the ED is caring for a school-age child who has epiglottis. Which of the following actions should the nurse take?

Monitor the child's O2 saturation The ​nurse ​should ​monitor ​the ​child's ​oxygen ​saturation ​level ​because ​the ​child ​is ​experiencing ​acute respiratory ​distress ​and ​it ​is ​necessary ​to ​determine ​if ​the ​child ​is ​responding ​to ​treatment.

A nurse is teaching a group of parents about infectious mononucleosis. Which of the following statements by a parent should the nurse identify as understanding the teaching

Mononucleosis is caused by an infection with the Epstein-Barr virus

Pharmacokinetics and Routes of Administration: Teaching Parents About the Administration of Eye Medications (Active Learning Template - Nursing Skill, RM FUND 9.0 Ch 46)

NURSING CONSIDERATIONS ●Use medical aseptic technique when instilling medications in eyes. ●Have clients sit upright or lie supine, tilt their head slightly, and look up at the ceiling. ●Rest your dominant hand on the clients' forehead, hold the dropper above the conjunctival sac about 1 to 2 cm, drop the medication into the center of the sac, avoid placing it directly on the cornea, and have them close the eye gently. If they blink during installation, repeat the procedure. ●Apply gentle pressure with your finger and a clean facial tissue on the nasolacrimal duct for 30 to 60 seconds to prevent systemic absorption of the medication. ●If instilling more than one medication in the same eye, wait at least 5 minutes between them. ●For eye ointment, apply a thin ribbon to the edge of the lower eyelid from the inner to the outer canthus

Gastrointestinal Disorders: Dietary Teaching for Celiac Disease (Active Learning Template - System Disorder, RM Nutrition 6.0 Chp. 13)

NURSING INTERVENTIONS ●Encourage clients to eat foods that are gluten‑free: milk, cheese, rice, corn, eggs, potatoes, fruits, vegetables, fresh meats and fish, dried beans. ●Remind clients to read labels on processed products. Gravy mixes, sauces, cold cuts, soups, and many other products have gluten as an ingredient. Advise clients to read labels and research nonfood products (e.g., lipstick, communion wafers, vitamin supplements), which also can have gluten as an ingredient.

A nurse is assessing an infant who has pneumonia. Which of the following findings is the priority for the nurse to report to MD?

Nasal flaring When ​using ​the ​airway, ​breathing, ​circulation ​approach ​to ​client ​care, ​the ​nurse ​should ​place ​the ​priority on ​nasal ​flaring. ​Nasal ​flaring ​indicates ​that ​the ​infant ​is ​experiencing ​acute ​respiratory ​distress

Dumping syndrome

Normally, the stomach controls the rate in which nutrients enter the small intestine. When a portion of the stomach is surgically removed, the contents of the stomach are rapidly emptied into the small intestine, causing dumpingsyndrome. ●Early manifestations typically occur 15 to 30 min after eating. Early manifestations include a sensation of fullness, abdominal cramping, nausea, diarrhea, and vasomotor manifestations (faintness, syncope, diaphoresis, tachycardia, hypotension, flushing). ●Late manifestations occur 1 to 3 hr after eating. Late manifestations include diaphoresis, weakness, tremors, anxiety, nausea, and hunger. ●Manifestations resolve after intestine is emptied. However, there is a rapid rise in blood glucose and increase in insulin levels immediately after the intestine empties. This leads to hypoglycemia.

Now the systems:

Now the systems:

A nurse is preparing to collect a sample from a toddler for a sickle-turbidity test. Appropriate action to take?

Perform a finger stick The ​nurse ​should ​perform ​a ​finger ​stick ​on ​a ​toddler ​as ​a ​component ​of ​the ​sickle-turbidity ​test. ​If ​the ​test is ​positive, ​hemoglobin ​electrophoresis ​is ​required ​to ​distinguish ​between ​children ​who ​have ​the ​genetic trait ​and ​children ​who ​have ​the ​disease.

A nurse is performing an abortion examination on a preschooler. Which of the following instructions should the nurse give to the child when performing abdominal palpitation

Place your hand under mine Allowing the child to touch her abdomen during the examination will promote relaxation.

Head Injury: Planning Care for an Infant Who Has an Epidural Hematoma (RN QSEN - Safety , Active Learning Template - System Disorder, RM NCC RN 10.0 Chp 14)

PATIENT‑CENTERED CARE nurSInG Care ●Care is determined by the extent of the brain trauma. ●Ensure the spine is stabilized until a spinal cord injury is ruled out. ●Monitor vital signs, level of consciousness, pupils, ICP, motor activity, sensory perception, and verbal responses at frequent intervals. Use the Glasgow Coma Scale as indicated. ●Maintain a patent airway. Provide mechanical ventilation as indicated. ●Administer oxygen as indicated to maintain an oxygen saturation level greater than 95%. ●Use padded restraints for clients who have agitation to prevent injury. ●Assess for clear fluid drainage from ears or nose (cerebral spinal fluid) and report to the provider. ●Assess for bleeding from the ear (indicates basal skull fracture) and report to the provider. ●Implement actions that will decrease ICP. ◯Keep the head midline with the bed elevated to 30°, which will also promote venous drainage. ◯Avoid extreme flexion, extension, or rotation of the head and maintain in midline neutral position. ◯Keep the client's body in alignment, avoiding hip flexion/extension. ◯Minimize oral suctioning. Nasal suctioning is contraindicated. ◯Instruct the client to avoid coughing and blowing the nose, because these activities increase ICP. ●Implement measures to prevent complications of immobility (turn every 2 hr, maintain footboard and splints). Specialty beds might be used. ●Insert and maintain an indwelling urinary catheter. Administer stool softener to prevent straining (Valsalvamaneuver). ●Provide a calm, restful environment (limit visitors, minimize noise). ●Use energy‑conservation measures, such as alternating activities with rest periods and cluster nursing activities ●Implement seizure precautions. ●Monitor fluid and electrolyte values and osmolarity to detect changes in sodium regulation, the onset of diabetes insipidus, or severe hypovolemia. ●Provide adequate fluids to maintain cerebral perfusion. When a large amount of IV fluids is prescribed, monitor the client for excess fluid volume, which might increase ICP. ●Maintain the client's safety (side rails up, padded side rails, call light within reach). ●Provide nutritional support (total parenteral nutrition, enteral nutrition). ●Maintain ongoing communication with the client and family. ●Instruct the family on effective ways to communicate with the child (touching, cuddling, talking, assisting with care as appropriate).

Gastrointestinal Structural and Inflammatory Disorders: Discharge Teaching Following Cheiloplasty (Active Learning Template - Therapeutic Procedure, RM NCC RN 10.0 Chp 23)

POSTOPERATIVE NURSING ACTIONS ●Perform standard postoperative care, including assessment of vital signs, oxygen saturation, and pain management using an age‑appropriate tool. ●Keep the infant pain‑free to decrease crying and stress on repair. ●Administer analgesics as prescribed. ●Assess operative sites for signs of crusting, bleeding, and infection. ●Avoid having the infant suck on a nipple or pacifier. ●Avoid spoons, forks, and other objects the infant might bring to her mouth that could damage the incision site. ●Monitor I&O and weigh daily. ●Observe the family's interaction with the infant. ●Assess family coping and support. ●For cleft lip ◯Monitor the integrity of the postoperative protective device to ensure proper positioning. ◯Position the infant on her back and upright, or on her side during the immediate postoperative period to maintain the integrity of the repair. ◯Apply elbow restraints to keep the infant from injuring the repair site. ◯Restraints should be removed periodically to assess skin, allow limb movement, and provide for comfort. ◯Use normal saline, water, or diluted hydrogen peroxide to clean the incision site. Apply antibiotic ointment if prescribed. ◯Gently aspirate secretions of mouth and nasopharynx to prevent respiratory complications. ●For cleft palate ◯Change the infant's position frequently to facilitate drainage and breathing. The infant may be placed on the abdomen in the immediate postoperative period. ◯Maintain IV fluids until the infant is able to eat and drink. ◯Clear liquid diet for first 24 hr. ◯Avoid placing a straw, tongue depressor, hard pacifier, rigid utensils, hard‑tipped sippy cups, or suction catheters in the infant's mouth after cleft palate repair. ◯Elbow restraints can needed to be used to prevent the infant from injuring the repair ◯Close observation for signs of airway obstruction, hemorrhage, and laryngeal spasm. ◯Face mask to deliver oxygen *CLIENT EDUCATION* ●Teach parents that the infant can require elbow restraints until lip/palate are healed. Instruct parents in the proper use of the restraints. ●Instruct parents on the postoperative diet and feeding techniques. ●Instruct parents in proper care of operative site

Hospitalization causes many issues - Stress is the Big one. This can be positive and negative, please explain. What is the child most afraid of - 3 things?

Painful procedure losing control what will happen to them object permanence = 9 months mile stone *web X* *stress from everyone happens after we come from the hospital bc we dont know all of the things thats going to happen or take place * for a child it is very different bc they dont understand as much and they do not know how to manage themselves when it comes to pain *most things they're afraid of is - loss of control/independence -bodily harm - pain: when we take something away they do not realize why were need to take it away, OR CUT INTO THEIR BODY *we can help with the anxiety and fear by---fear increases itself in pain perception. We can decrease that pain perception by taking some of that away* have times explaining what they understand have their parents with them

A nurse is caring for a school-age child who has peripheral edema. Following assessments that confirm peripheral edema?

Palpate the dorsum of the child's feet The ​nurse ​should ​palpate ​the ​dorsum ​of ​the ​feet ​by ​pressing ​her ​fingertip ​against ​a ​bony ​prominence ​for 5 ​seconds ​to ​assess ​for ​peripheral ​edema.

A nurse in an emergency department suspects that a toddler has epiglottitis. Which of the following actions should the nurse take

Prepare the toddler for nasotracheal intubation

A nurse is assessing a 6-month-old infant at a well-infant visit. Which of the following findings should the nurse report to the provider

Presence of strabismus

A nurse is creating a plan of care for a school-age child who has heart disease and has developed heart failure. Which of the following interventions should the nurse include in the plan

Provide small, frequent meals to the child

A nurse is planning developmental activities for a newly admitted 10-year-old child who has neutropenia. Which of the following actions should the nurse plan to take?

Provide the child with a book about adventure. The ​nurse ​should ​provide ​a ​school-age ​child ​with ​a ​book ​about ​adventure ​as ​a ​developmental ​activity because ​children ​are ​expanding ​their ​knowledge ​and ​imagination ​during ​this ​age. ​Through ​reading, school-age ​children ​can ​feel ​powerful ​and ​skillful ​as ​they ​imagine ​themselves ​in ​the ​stories ​they ​read.

Chronic Neuromusculoskeletal Disorders: Assessment of Spastic Cerebral Palsy (Active Learning Template - System Disorder, RM NCC RN 10.0 chp 29)

RN NURSING CARE OF CHILDRENCHAPTER 29ChroniCneuromusCuloskeletal DisorDers181UNIT 2NURSING CARE OF CHILDREN WHO HAVE SYSTEM DISORDERSSECTION:MUSCULOSKELETAL DISORDERSCHAPTER 29 Chronic Neuromusculoskeletal DisordersChronic problems associated with mobility can be reflective of a problem with the musculoskeletal system or the result of a disorder related to the neural pathway extending from the brain's cortex to the neuromuscular junction.Cerebral palsy ●Cerebral palsy (CP) is a nonprogressive impairment of motor function, especially that of muscle control, coordination, and posture. ●CP can cause abnormal perception and sensation; visual, hearing, and speech impairments; seizures; and cognitive disabilities. ●CP manifests differently in each child. Developmental outcomes vary and are dependent on the severity of t he i njur y. PHYSICAL ASSESSMENT FINDINGS ●Failure to meet developmental milestones ●Persistent primitive reflexes (Moro or tonic neck) ●Gagging or choking with feeding, poor suck reflex ●Tongue thrust ●Poor head control ●Rigid posture and extremities, abnormal posturing ●Asymmetric crawl ●Hyperreflexia ●Vision, speech, or hearing impairments ●Seizures ●Impaired social relationships

RN Nursing Care of Children Online Practice 2016 B

RN Nursing Care of Children Online Practice 2016 B

RN Nursing Care of Children Online Practice 2016 B remediation

RN Nursing Care of Children Online Practice 2016 B remediation

A nurse is caring for a newly-admitted school-age child who has hypopituitarism. Which of the following medications should the nurse expect the provider to recommend to the parents for treating the child's condition

Recombinant growth hormone

A nurse is documenting findings from a physical examination. Which of the following statements indicates correct charting?

Regular heart rate and rhythm: S1, S2 heard. This is clear, concise charting for heart sounds.

A nurse is assessing a 3-yo toddler at a well-child visit. Which of the following manifestations should the nurse report to the MD?

Respiratory rate 45/min A ​respiratory ​rate ​of ​45/min ​is ​above ​the ​expected ​reference ​range ​for ​a ​3-year-old ​toddler ​and ​can indicate ​respiratory ​dysfunction ​and ​acute ​respiratory ​distress. ​Therefore, ​the ​nurse ​should ​report ​this finding ​to ​the ​provider ​immediately.

A nurse is providing discharge teaching to the parents of a Caucasian toddler who had a lower leg cast applied 24 hr ago. The nurse should instruct the parents to report which of the following findings to the provider

Restricted ability to move the toe

Planning care for a toddler who has a serum lead leverl of 4 mcg/dL. Appropriate action to take?

Schedule the toddler for a yearly rescreening. The ​nurse ​should ​schedule ​the ​toddler ​for ​a ​lead ​level ​rescreening ​in ​1 ​year ​and ​educate ​the ​family ​on ways ​to ​prevent ​exposure. Chelation ​therpay ​is ​required ​for ​lead ​level ​of ​45, ​or ​can ​be ​intiated ​with ​lead ​levels ​above ​10

A nurse is caring for a school-age child who is receiving chemotherapy and is severely immunocompromised. Which of the following actions should the nurse take

Screen the child's visitors for indications of infection

A nurse is caring for an adolescent who received a kidney transplant. Which of the following findings should the nurse identify as an indication the adolescent is rejecting the kidney

Serum creatinine 3.0 mg/dt

Renal issue s- UTI are very popular among children but may manifest them differently: what are some of the signs of a UTI in a child? Why are children more prone to UTI's

Short urethra dirty hands, bacteria poor hygiene pee after sex **vague symptomology .. tummy ache. back pain fever abd n/v. vague pain = check the urine darn busy bees playing too much and not eating or drinking any good nutrients... darn hydration !!! the best way to identify with children is daily weights... this can show fluid retention.. check for renal health.. Glomero .. NOT CHECKING FOR EDEMA *web x* we tx acute and chronic renal failure through dialysis peritoneal dialysis works better for kids (more beneficial for the kids) some kids get hemodialysis but that means they are locked in a machine for hours 7 times a week ----------------- uti's kids do not present the same as adults very young children and old adults have very vague symptoms may have behavioral/mentation issues pain in the belly/back some blood/ irritability lethargy or gargled speech

A nurse is reviewing the lab report of an infant who is receiving tx for severe dehydration. The nurse should identify that which of the following lab values indicates effectiveness of the current tx?

Sodium 140 mEq/L The ​nurse ​should ​identify ​that ​a ​sodium ​level ​of ​140 ​mEq/L ​is ​within ​the ​expected ​reference ​range ​and indicates ​the ​current ​treatment ​regimen ​the ​infant ​is ​receiving ​for ​dehydration ​is ​effective.

Asthma - complex respiratory illness - how do we recognize and how do we manage and educate the parents.

Sx - wheezing, stridor, SOB, nasal flairng assessory muscles bronchodiolators, steriods, O2 (always use nasal cannula first) Educate about the Triggers increase humidity daily flow meter.. DO DAILY MEDICINE *web x* kids get upper and lower respiratory infections bc of - shorter softer airway - short trachea - shorter Eustachian tubes - exposed to more bacteria (play dates) -soft epiglottis - large tonsils - weaker immune system - dont wash there hands and evrything goes into their mouth - *Upper respiratory infections (URI) easily nursing interventions is fluid replacement watching there nutritional intake and rest* - with asthma etc look at birth history and see if they were premature---sometimes it is from smoking families or just occurs -*give meds that are going to open the airway like bronchodilaters, misters, airway tx, humidafiers(cold) not hot/ steam bc kids can get burned, vix vaper rub* if giving iv meds make sure iv is patent children have a tendency to become infiltrated and abx in tissue is toxic and will burn the tissue

Know the risks of Croup syndromes - and how to manage.

Sx barking sound Humidify air best treatment *web x* barking sound -open/ maintain the airway -elevate the head of the bed (high fowlers, or tripod position) - cool/warm moist (shower) - gets worse at evening/night - wet/cool weather bring them back inside -steroids - no cold outside air -droplet precautions for 24 hr

A charge nurse in an ED is preparing an in-service for a group of newly licensed nurses on the clinical manifestations of child maltreatment. Which of the following clinical manifestations should the charge nurse include as suggestive of potential physical abuse?

Symmetric burns of the lower extremities The ​nurse ​should ​include ​in ​the ​teaching ​that ​symmetric ​burns ​of ​the ​lower ​extremities ​are ​a ​suggestive clinical ​manifestation ​of ​physical ​abuse. ​The ​patterns ​are ​usually ​characteristic ​of ​the ​method ​or ​object used, ​such ​as ​cigar ​or ​cigarette ​burns, ​or ​burns ​in ​the ​shape ​of ​an ​iron.

Benefits of play

TEMPERMENT does not change intellectual development, learn, consequence toddlers are onlooker ego centric babys = solotary toddler parrel play preschool - associate school age - organized if onlooker play prolongs this is a red flag *web x* learn to socialize learn to communicate learn their language learn there own strength fine motor skills play does help mentally and dev. not much physically they learn to observe, share, and interact with each other *Temperament does not change* - we can influence how we respond to someone's temperament and will cause children to change some of there behaviors - Temperament you're born with it, it deals with genetics - those who are difficult, easy, slow to warm * some are going to be hesitant all there life, sit back and watch others and are hesitant that they just dont jump right in- these are the SLOW TO WARM * easy going people who go with the flow (these kids do well in a hospital setting* difficult Temperament do not do as well bc these kids dont like change and they dont like having to re-learn things, so they tend to struggle more with the stress level children with rural setting have more difficulty children with a disability do better in a hospital setting bc they are comfy with having that type of environment

A nurse is auscultating the lungs of an adolescent who has asthma. The nurse should identify the sound as which of the following?

Tachypnea abnormally rapid breathing. The ​nurse ​should ​identify ​the ​sound ​heard ​during ​auscultation ​as ​tachypnea, ​which ​is ​a ​rapid, ​regular breathing ​pattern. ​This ​breathing ​pattern ​often ​occurs ​with ​anxiety, ​fever, ​metabolic ​acidosis, ​or ​severe anemia.

A nurse is assessing the vital signs of a 10-year-old child following a burn injury. Clinical manifestation that indicate early septic shock?

Temperature 39.1C​ ​(102.4​F) The ​nurse ​should ​expect ​a ​child ​who ​has ​early ​septic ​shock ​to ​have ​a ​fever ​and ​chills.

Physical Assessment Findings: Reportable Findings for a 3-Year-Old Child (Active Learning Template - Growth and Development, RM NCC RN 10.0 Chp 2)

Temperature by age 3 YEARS37.2 ̊ c (99.0 ̊ F) ●axillary ●tympanic ●Oral (if child cooperative) ●Rectal (if exact measurement necessary PUlse Rate 2 to 10 years: 60 to 110/min (depending on activity ResPiRatiOns 2 to 6 years: 21 to 25/min BlOOd PRessURe ●Readings should be compared with standard measurements (National High Blood Pressure Education Program Working Group on High Blood Pressure in Children and Adolescents). ●Age, height, and gender all influence blood pressure readings

A nurse is assessing an adolescent who received a sodium polystryrene sulfonate enema. Which of the following findings indicates effectiveness of the medication.

The adolescent's serum potassium level is 4.1 mEq/L The ​nurse ​should ​monitor ​the ​adolescent's ​serum ​potassium ​level ​following ​the ​administration ​of ​sodium polystyrene ​sulfonate. ​This ​medication ​is ​used ​to ​treat ​hyperkalemia ​by ​exchanging ​sodium ​ions ​for potassium ​ions ​in ​the ​intestine. ​Therefore, ​a ​potassium ​level ​within ​the ​expected ​reference ​range indicates ​the ​effectiveness ​of ​the ​medication.

A nurse is planning an educational program for school-age children and their parents about bicycle safety. Which of the following information should the nurse plan to include

The child should be able to stand on the balls of her feet when sitting on the bike

A nurse is teaching a school-age child who has a severe allergy to bee venom and his parent about epinephrine. Which of the following instructions should the nurse include in the teaching

Use a second dose if the first dose of epinephrine does not completely reverse the symptom

A nurse is planning care for a school-age child who has a tunneled central venous access device. Which of the following interventions should the nurse include in the plan

Use a semipermeable transparent dressing to cover the site

Cystic fibrosis - is very complex and affects many systems - can you list them and how do we manage them, and educate the parents.

This is in 2 or more systems can causes over production of enzymes.. MUCUS EVERY WHERE. biggest issue can be in lungs from this Sx - clubbing of extremities. malnourished small thin, pale, lethargy Tx - both parents of recessive gene. person with it is sterile. ... small frequent meals, medication, inhalers, chest breathing therapy all the time, enzyme for pancreatic 30min before meals. may reach middle adulthood. sodium, salt on skin =chloride test. sodium loss. most have lung transplant *web x* respiratory is lst, but does effect these other systems and we need to manage it multi system problem- exocrine dz, so there is mucus throughout the body this is genetic alot of mucus throughout the body in the lungs in the GI tract and in the reproductive tract/pancreas kids will be small and cant participate in a lot of activities - needs alot of rest - needs alot of o2 support - fluids carefully - high caloric diet with low volume, they cannot eat great volumes of food but they need alot of calories bc they're not going to absorb the food -low fat diet - add pancreatic enzymes to everything - men will be sterile bc the sperm does not come through the tract and it is all mucoid - women the eggs are not going to get through the ovarian tract whole total body problem doesnt live beyond the age of 22-25 yr without a transplant nucleus ileus is an early symptom so we should be thinking about this if a baby has not passed meconium in 24 hr very fatty stools causing the stool to float and they have a salty taste to their skin bc they have all the na cl in their system

Endocrine issues - thyroid, diabetes, etc - remember that children should be in charge of their care where appropriate. Know the signs of each endocrine issue,

Thyroid Sx & Tx should be the same for adult and children.. Sx show differently in children tho. might be withdrawn.. Type 1 diabetes.. poor kids : ( --- 3 P's poly uria, polyphaisa, poly dipseia.. losing weight. fruity breath.. Cognitive changes.. Children at risk for Kitoacidoas AKA -- teach children to manage their health as much as possible!!!!!! can not route insuline inject shot location as much... children will hopefully be on long acting or pump. Kids can particiapte in all activities.. more protien, snack, fluid If not growing. give growth hormone at night HS because that is when it occurs naturally.

Surgical interventions for the children: Repair of lacerations, tonsillectomy, appendectomy. What are the nursing interventions that should be managed in most post-operative cases?

Tonsillectomy .. post op - check swallowing, pain, no dairy, no citrus, no color, clear liquids safe, check LOC, no straws, last void, don't suction Appendectomy no laxatives.. pass gas, check incision, ambulate if reasonable "soon" not immediate, LOC, PAIN Assessment FACEs PAIN Assessment Cry scale = babies Toddler = FLACC scale Wong -Baker/ Faces After 10 to 12years numeric, all nursing staff use save evaluation *web x* the key for sx intervention is tytration meds and o2 do a thorough assessment post op in these kids bc they do not know what to tell you and neither does parents listen to the heart, lungs, belly look at the incision where is the iv do they have o2 when is the last time they voided do they have sensation in all extremities anesthesia may move off of them quicker or slower bc they are not able to metabolize it as easily give opiods if its a sx intervention- we want to be careful on how we give the opiods and how much WE DONT GIVE ASPIRIN BC OF RAYS SYNDROME

What are signs of Respiratory distress in babies - and then other children

Tripod position !!! trying to get some air Epiglotitios is the most severs respiratory issue

A nurse is caring for a preschooler who has been receiving IV fluids via a peripheral IV catheter. When preparing to discontinue the IV fluids and catheter, which of the following actions should the nurse plan to take?

Turn of the IV pump Occlude the IV tubing Remove the tape securing the catheter Apply pressure over the catheter insertion site

Which of the following techniques is appropriate when obtaining a blood pressure on a child?

Use a cuff with bladder covering 80 to 100% of the arm circumference

Communicable Diseases: Responding to a Parent About Varicella (RN QSEN - Patient-centered Care, Active Learning Template - System Disorder, RM NCC RN 10.0 chp 36)

Varicella (chickenpox)/ varicella‑zoster virus SPREAD ●Direct contact ●Droplet (airborne) ●Contaminated objects INCUBATION:2 to 3 weeks, usually 14 to 16 days COMMUNICABILITY:1 day before lesions appear until all lesions have formed crusts

Injection sites

Vasta lateralis (thigh) = babies !!! Ventral Gluteal (butt) = landmarks are easy to find. might be better then deltoid Shortest catheter, short needles. Get help! get distractions. have someone with you.. ALWAYS USE PROCEDURE ROOM

A nurse in a provider's office is preparing to administer immunizations to a toddler during a well-child visit. Which of the following actions should the nurse plan to take?

Withhold the influenza vaccine Exhibit ​1: ​Tuberculin ​skin ​test, ​Measles, ​mumps, ​rubella ​vaccine, ​live ​attenuated ​influenza ​vaccine, ​varicela vaccine Exhibit ​2: ​rr ​24/min, ​HR ​115/min, ​temp ​37.4/99.3F, Exhibit ​3: ​age ​12 ​months ​9 ​days, ​71.1 ​cm. ​Iodine ​allergy, ​Hx ​of ​asthma The ​live ​attenuated ​influenza ​vaccine ​is ​contraindicated ​in ​a ​child ​who ​has ​asthma.

Burns: Wound Debridement Following a Burn Injury (Active Learning Template - Therapeutic Procedure, RM NCC RN 10.0 chp 32)

Wound care NURSING ACTIONS ●Premedicate as prescribed prior to wound care. ◯Administer analgesics. ◯Administer hydroxyzine or diphenhydramine for pruritus. ●Remove previous dressings. ●Assess for odors, drainage, and discharge. ●Cleanse the wound as prescribed. ●Assist with debridement. ◯Provide hydrotherapy (place client or affected extremity in a warm tub of water or use warm running water, as if to shower) to cleanse the wound. Use once or twice a day. Use mild soap or detergent to gently wash burns, and then rinse with tepid water. ◯Encourage active range of motion during hydrotherapy .◯Monitor for cold stress and hypothermia

A nurse is teaching the mother of a 6-month-old infant about teething. Which of the following statements should the nurse make

Your baby may pull at her ears when she is teething

A nurse is providing teaching to the parent of an infant who has diaper dermatitis. The nurse should teach the parent to apply which of the following to the affected area

Zinc oxide

cognitive issues

speak to child at the level they understand * might be 21, but understands at the level of a 14 yr

Consents - for invasive procedures - for children who is responsible to sign consents and what is important to know about them.

stay with the pt and check q so often dont need consent for circumcision (papoose board) no consent for an elbo arm for someone with an iv --------------- legal guardian or parent someone over the age of 18 emancipated adult someone who is pregnant court order now they are able to speak for themselves have to understand the language 2 nurses to document that they made every legal attempt to contact the legal guardian or parent in case of emergency have to understand the procedure have to understand the benefits and the risk have to voluntary accept the procedure and the parents are doing that for them, saying yes they volunteer they're not being coarse dr gives the info and the nurses witnesses the signature nurse can re iterate the info, but were are not suppose to initiate the conversation about consent about procedured and meds required

GI issues - dehydration is huge. Children that become dehydrated may suffer long term organ damage - so we must encourage fluids - PO or IV. Watch the titration however, it may cause problems if we fun fluids too rapidly. (such as) We do not restrict fluids, but should be mindful of what types of things they are drinking - milk may thicken secretions - fruit juice may induce diarrhea, soda/pop - may also induce diarrhea or cramping - so we must think that through when parents are trying to get them to drink.

sugar causes gas etc. MOST NUTRIENTS ARE ABSORBED IN THE SMALL INTESTINES *web x* children do not handle shifts of fluid or temperature well if kids get severly dehydrated it causes acute renal failure we need to recognize this very quickly if parents cannot get them to drink we need to intervene and give iv therapy and pump fluids back in carefully and titrate carefully bc we need to get the fluids back in to get the electrolytes back in *when re hydrating* stay away from dairy juices bc of sugar caffeine *with babies we will see a depressed fontanel with children we will see dry mucus membrane no tears when crying they become so lethargic that u stick th em with a 100 pins and they never flinch ( this is a severely dehydrated child) we want to make sure we can get the fluids in

A nurse is assessing a toddler who has gastroenteritis and is exhibiting manifestations of dehydration. Which of the following findings should the nurse address first

tachypnea

A nurse is performing an annual physical examination on an adolescent. Which of the following should be included in the general survey?

the patient makes good eye contact This information is included in the general survey. The survey includes identifying the patients demeanor, mood and interactions with others.

A nurse is preparing to administer a hep B vaccine to a 1-month-old. The nurse should plan to inject the medication at which location?

thigh

A nurse is assessing a toddler who has leukemia and is receiving his first round of chemotherapy. Which of the following findings is the priority for the nurse to report to the provide

urticaria

A nurse is admitting an infant who has intussusception. Which of the following findings should the nurse expect? (Select all that apply

vomiting, lethargy

A nurse in an emergency department is auscultating the lungs of an adolescent who is experiencing dyspnea. The nurse should identify the sound as which of the following? (Click on the audio button to listen to the clip

wheezes

Dumping syndrome NURSING INTERVENTIONS

●Recommend small, frequent meals. ●Recommend consuming protein and fat at each meal. ●Tell the client to avoid food that contains concentrated sugars and to restrict lactose intake. ●Suggest that the client consume liquids 1 hr before or after eating instead of during meals (dry diet). ●Instruct client to lie down for 20 to 30 min after meals to delay gastric emptying. If reflux is a problem, suggest a reclining position. ●Monitor clients receiving enteral tube feedings and report manifestations of dumping syndrome to the provider. ●Monitor the client for vitamin and mineral deficits, such as iron and vitamin B12

Acute Infectious Gastrointestinal Disorders: Evaluating Understanding of Dehydration (Active Learning Template - System Disorder, RM NCC RN 10.0 Chp 22)

◯MANIFESTATIONS ■Capillary refill greater than 4 seconds ■Tachycardia present, and orthostatic blood pressure can progress to shock ■Extreme thirst ■Very dry mucous membranes and tented skin ■Hyperpnea ■No tearing with sunken eyeballs ■Sunken anterior fontanel ■Oliguria or anuria


Ensembles d'études connexes

Personal Finance Quizzes CH 1, 2, 4, 5, 6, 7

View Set

Unit 10: Defaults and Foreclosures

View Set

Angličtina B2 (part1) (CZ/EN) by TASMAN

View Set

ART OF LISTENING 7 Elements of music

View Set

How the Universe Works: Big Bang

View Set